Christine has a motion detector light which gets activated an average of 16 times every 2 hours during the night. In order to find the probability that the motion detector light will be activated more than 4 times in a 25 minute period during the night using the Poisson distribution, what is the average number of activations per 25 minutes?

Answers

Answer 1

Answer:

The average number of activations per 25 minutes is 2.5.

Step-by-step explanation:

In a Poisson distribution, the probability that X represents the number of successes of a random variable is given by the following formula:

[tex]P(X = x) = \frac{e^{-\mu}*\mu^{x}}{(x)!} [/tex]

In which

x is the number of sucesses

e = 2.71828 is the Euler number

[tex]\mu[/tex] is the mean in the given interval.

In this question:

For 2 hours = 2*60 = 120 minutes, the mean is 16 times.

To find for 25 minutes, we use a rule of three.

120 minutes - 16 times

25 minutes - m times

[tex]120m = 12*25[/tex]

[tex]m = \frac{12*25}{120}[/tex]

[tex]m = 2.5[/tex]

The average number of activations per 25 minutes is 2.5.


Related Questions

A toy manufacturer wants to know how many new toys children buy each year. Assume a previous study found the standard deviation to be 1.8. She thinks the mean is 5.8 toys per year. What is the minimum sample size required to ensure that the estimate has an error of at most 0.12 at the 80% level of confidence

Answers

Answer:

[tex]n=(\frac{z_{\alpha/2} \sigma}{ME})^2[/tex]   (b)

The critical value for 80% of confidence interval now can be founded using the normal distribution the significance level would be 20% and the critical value [tex]z_{\alpha/2}=1.28[/tex], replacing into formula (b) we got:

[tex]n=(\frac{1.28(1.8)}{0.12})^2 =368.64 \approx 369[/tex]

So the answer for this case would be n=369 rounded up to the nearest integer

Step-by-step explanation:

We know the following info given:

[tex] \sigma = 1.8[/tex] represent the standard deviation

[tex]\mu = 5.8[/tex] the true mean that she believes

[tex] ME = 0.12[/tex] represent the margin of error

The margin of error is given by this formula:

[tex] ME=z_{\alpha/2}\frac{s}{\sqrt{n}}[/tex]    (a)

And on this case we have that ME =+0.12 and we are interested in order to find the value of n, if we solve n from equation (a) we got:

[tex]n=(\frac{z_{\alpha/2} \sigma}{ME})^2[/tex]   (b)

The critical value for 80% of confidence interval now can be founded using the normal distribution the significance level would be 20% and the critical value [tex]z_{\alpha/2}=1.28[/tex], replacing into formula (b) we got:

[tex]n=(\frac{1.28(1.8)}{0.12})^2 =368.64 \approx 369[/tex]

So the answer for this case would be n=369 rounded up to the nearest integer

A consumer affairs investigator records the repair cost for 4 randomly selected TVs. A sample mean of $91.78 and standard deviation of $23.13 are subsequently computed. Determine the 90% confidence interval for the mean repair cost for the TVs. Assume the population is approximately normal. Step 1 of 2 : Find the critical value that should be used in constructing the confidence interval. Round your answer to three decimal places.

Answers

Answer:

= ( $72.756, $110.804)

Therefore, the 90% confidence interval (a,b) = ( $72.756, $110.804)

Critical value at 90% confidence = 1.645

Step-by-step explanation:

Confidence interval can be defined as a range of values so defined that there is a specified probability that the value of a parameter lies within it.

The confidence interval of a statistical data can be written as.

x+/-zr/√n

Given that;

Mean x = $91.78

Standard deviation r = $23.13

Number of samples n = 4

Confidence interval = 90%

Using the z table;

z(α=0.05) = 1.645

Critical value at 90% confidence = 1.645

Substituting the values we have;

$91.78+/-1.645($23.13/√4)

$91.78+/-1.645($11.565)

$91.78+/-$19.024425

$91.78+/-$19.024

= ( $72.756, $110.804)

Therefore, the 90% confidence interval (a,b) = ( $72.756, $110.804)

The probability of event A is 0.48, the probability of event A and B is 0.21, and the probability of events A or B is 0.89. What is the probability of event B? THE ANSWER IS 0.62

Answers

Answer:

P(B) = 0.62

Step-by-step explanation:

P(A or B) = P(A) + P(B) - P(A and B)

So, Putting the givens

0.89 = 0.48 + P(B) - 0.21

0.89 = 0.27 + P(B)

P(B) = 0.89 - 0.27

P(B) = 0.62

Aurora saved $850. Ahe used 35% of her savings on a new TV. How much did the TV cost?

Answers

Multiply her savings by the percent spent:

850 x 0.35 = 297.50

The tv cost $297.50

Answer:

the price of TV is = 297.5$

Step-by-step explanation:

all money= 850$

purchased money= 35% of all money ==> 850 ( 35%) = 297.5$

Using the definition of the​ derivative, find f prime (x ). Then find f prime (1 )​, f prime (2 )​, and f prime (3 )when the derivative exists.

Answers

Step-by-step explanation:

We need the function f(x) to be able to determine the required.

Suppose we were given a function

f(x) = y

f'(x) represents the first derivative of the function f(x) = y.

f'(1) represents the value of the first derivative of the function f(x) = y after replacing x by 1.

f'(5) represents the value of the first derivative of the function f(x) = y after replacing x by 5.

Example: Suppose f(x) = x² + 3x, find

f'(x), f'(1), and f'(5).

f'(x) = 2x + 3

f'(1) = 2(1) + 3 = 5

f'(5) = 2(5) + 3 = 13

What’s the correct answer for this?

Answers

Answer:

D: <K = 35°

Step-by-step explanation:

<E = 55

<L = 90°

Now

<LKE = 180-90-55

<K = 35°

Answer:

[tex]\fbox{\begin{minipage}{8.8em}Option D is correct\end{minipage}}[/tex]

Explanation:

Here, we state again the definition of inscribed angle in circle:

(1) An inscribed angle has the vertex on the circle and the sides are chords.

=> In the picture shown, angle ELK is inscribed angle with vertex L and LE and LK are chords.

(2)An inscribed angle also creates an intercepted arc whose endpoints are on the angle.

=> Inscribed angle ELK creates intercepted arc EK.

(3) According to the Inscribed Angle Theorem, the measure of intercepted arc is twice as the measure of its inscribed angle.

=> Angle ELK = (1/2) arc EK

Arc EK, whose EK is diameter, is equal to measure of half of circle, or 180 degree, in other words.

=> Angle ELK = (1/2) x 180 = 90 deg

(4) As the property of sum of 3 angles inside a triangle, this sum is equal to 180 degree.

=> Considering triangle ELK:

ELK + LEK + LKE = 180 deg

or

90 + 55 + LKE = 180 deg

or

LKE = 180 - 90 - 55 = 35 deg

Hope this helps!

:)

Method 1: Long Division (x^2+3x-43) / (x+8

Answers

Answer:

x - 5 - (3/x+8)

Step-by-step explanation:

Answer:

Step-by-step explanation:

If a tank holds 4500 gallons of water, which drains from the bottom of the tank in 50 minutes, then Toricelli's Law gives the volume V of water remaining in the tank after t minutes as V = 4500 1 − 1 50 t 2 0≤ t ≤ 50. Find the rate at which water is draining from the tank after the following amounts of time.a) 5 min 855 x gal/min b) 10 min 160 x gal/min c) 20 min 120 x gal/min d) 50 min gal/min

Answers

Answer:

a) at 5 minutes: 162 gal/min

b) at 10 minutes:  144 gal/min

c) at 20 minutes:  108 gal/min

d) at 50 minutes: 0 gal/min

Step-by-step explanation:

Considering the formula given by the volume of water remaining in the tank:

[tex]V(t)=4500\,(1-\frac{1}{50} \,t)^2[/tex]

we can find the rate of water draining from the tank, (that is change in volume divided elapsed time) with the derivative of the function at the different times. Notice that this function has a decaying curvature (see attached image) of volume as a function of time, and the idea is therefore to find the slope of the tangent line at the different requested times.

So we first calculate the derivative of this function at any time 't":

[tex]V(t)=4500\,(1-\frac{1}{50} \,t)^2\\V'(t)=9000\,(1-\frac{1}{50} \,t)\,(-\frac{1}{50})\\V'(t)=-180(1-\frac{1}{50} \,t)\\V'(t)=-180+3.6\,t[/tex]

And now we estimate this derivative at the different requested points for time values:

a) at 5 minutes:  [tex]V'(5)=-180+3.6\,(5) = -162\,\,gal/min[/tex]

b) at 10 minutes:   [tex]V'(10)=-180+3.6\,(10) = -144\,\,gal/min[/tex]

c) at 20 minutes:   [tex]V'(20)=-180+3.6\,(20) = -108\,\,gal/min[/tex]

d) at 50 minutes:  [tex]V'(50)=-180+3.6\,(50) = 0\,\,gal/min[/tex]

All the negative signs preceding indicate that the remaining volume in the tank is reducing as time goes by, so the volume at which the water is draining is actually the absolute value of those numbers.

Need Help!...anyone!

Answers

(a)

[tex] \sqrt[5]{ {x}^{3} } [/tex]

(b)

[tex] \sqrt[8]{x} [/tex]

(c)

[tex] \sqrt[3]{ {x}^{5} } [/tex]

(d)

[tex] \sqrt{ {x}^{3} } [/tex]

What is the value of the discriminant for the quadratic equation?
6x^2 - 2x + 5 = 0

Answers

Answer: -116 is value of discriminant

4. The average annual income of 100 randomly chosen residents of Santa Cruz is $30,755 with a standard deviation of $20,450. a) What is the standard deviation of the annual income? b) Test the hypothesis that the average annual income is $32,000 against the alternative that it is less than $32,000 at the 10% level. c) Test the hypothesis that the average annual income is equal to $33,000 against the alternative that it is not at the 5% level. d) What is the 95% confidence interval of the average annual income?

Answers

Answer:

a) The standard deviation of the annual income σₓ = 2045

b)

The calculated value Z = 0.608 < 1.645 at 10 % level of significance

Null hypothesis is accepted

The average annual income is greater than $32,000

c)

The calculated value Z = 1.0977 < 1.96 at 5 % level of significance

Null hypothesis is accepted

The average annual income is  equal to  $33,000

d)

95% of confidence intervals of the Average annual income

(26 ,746.8 ,34, 763.2)

Step-by-step explanation:

Given size of the sample 'n' =100

mean of the sample x⁻ =  $30,755

The Standard deviation = $20,450

a)

The standard deviation of the annual income σₓ = [tex]\frac{S.D}{\sqrt{n} }[/tex]

                                               = [tex]\frac{20,450}{\sqrt{100} }= 2045[/tex]

b)

Given mean of the Population μ =  $32,000

Given size of the sample 'n' =100

mean of the sample x⁻ =  $30,755

The Standard deviation ( σ)= $20,450

Null Hypothesis:- H₀: μ > $32,000

Alternative Hypothesis:H₁: μ <  $32,000

Level of significance α = 0.10

[tex]Z = \frac{x^{-}-mean }{\frac{S.D}{\sqrt{n} } }[/tex]

[tex]Z = \frac{30755-32000 }{\frac{20450}{\sqrt{100} } }[/tex]

Z= |-0.608| = 0.608

The calculated value Z = 0.608 < 1.645 at 10 % level of significance

Null hypothesis is accepted

The average annual income is  greater than $32,000

c)

Given mean of the Population μ =  $33,000

Given size of the sample 'n' =100

mean of the sample x⁻ =  $30,755

The Standard deviation ( σ)= $20,450

Null Hypothesis:- H₀: μ =  $33,000

Alternative Hypothesis:H₁: μ ≠ $33,000

Level of significance α = 0.05

[tex]Z = \frac{x^{-}-mean }{\frac{S.D}{\sqrt{n} } }[/tex]

[tex]Z = \frac{30755-33000 }{\frac{20450}{\sqrt{100} } }[/tex]

Z = -1.0977

|Z|= |-1.0977| = 1.0977

The 95% of z -value = 1.96

The calculated value Z = 1.0977 < 1.96 at 5 % level of significance

Null hypothesis is accepted

The average annual income is equal to  $33,000

d)

95% of confidence intervals is determined by

[tex](x^{-} - 1.96 \frac{S.D}{\sqrt{n} } , x^{-} + 1.96 \frac{S.D}{\sqrt{n} })[/tex]

[tex](30755 - 1.96 \frac{20450}{\sqrt{100} } , 30755 +1.96 \frac{20450}{\sqrt{100} })[/tex]

( 30 755 - 4008.2 , 30 755 +4008.2)

95% of confidence intervals of the Average annual income

(26 ,746.8 ,34, 763.2)

Assume a simple random sample of 10 BMIs with a standard deviation of 1.186 is selected from a normally distributed population of recent Miss America winners. Use 0.01 significance level to test the claim that the BMI for recent Miss America winners are from a population with standard deviation of 1.34.
A. Identify the null hypothesis and the alternative hypothesis.
B. Find the critical value or values.
C. Find the test statistic.
D. State the conclusion that addresses the original claim.

Answers

Answer:

a) H0: [tex]\sigma = 1.34[/tex]

H1: [tex]\sigma \neq 1.34[/tex]

b) [tex] df = n-1= 10-1=9[/tex]

And the critical values with [tex]\alpha/2=0.005[/tex] on each tail are:

[tex] \chi_{\alpha/2}= 1.735, \chi_{1-\alpha/2}= 23.589[/tex]

c) [tex] t=(10-1) [\frac{1.186}{1.34}]^2 =7.05[/tex]

d) For this case since the critical value is not higher or lower than the critical values we have enough evidence to FAIL to reject the null hypothesis and we can conclude that the true deviation is not significantly different from 1.34

Step-by-step explanation:

Information provided

n = 10 sample size

s= 1.186 the sample deviation

[tex]\sigma_o =1.34[/tex] the value that we want to test

[tex]p_v [/tex] represent the p value for the test

t represent the statistic  (chi square test)

[tex]\alpha=0.01[/tex] significance level

Part a

On this case we want to test if the true deviation is 1,34 or no, so the system of hypothesis are:

H0: [tex]\sigma = 1.34[/tex]

H1: [tex]\sigma \neq 1.34[/tex]

The statistic is given by:

[tex] t=(n-1) [\frac{s}{\sigma_o}]^2 [/tex]

Part b

The degrees of freedom are given by:

[tex] df = n-1= 10-1=9[/tex]

And the critical values with [tex]\alpha/2=0.005[/tex] on each tail are:

[tex] \chi_{\alpha/2}= 1.735, \chi_{1-\alpha/2}= 23.589[/tex]

Part c

Replacing the info we got:

[tex] t=(10-1) [\frac{1.186}{1.34}]^2 =7.05[/tex]

Part d

For this case since the critical value is not higher or lower than the critical values we have enough evidence to FAIL to reject the null hypothesis and we can conclude that the true deviation is not significantly different from 1.34

What is 2 1/2 + 1 1/3

Answers

Answer:

[tex]=3\frac{5}{6}[/tex]

Step-by-step explanation:

[tex]2\frac{1}{2}+1\frac{1}{3}\\\mathrm{Add\:whole\:numbers}\:2+1:\quad 3\\\mathrm{Combine\:fractions}\:\frac{1}{2}+\frac{1}{3}:\quad \frac{5}{6}\\=3\frac{5}{6}[/tex]

how many real solutions does the equation x2 − 9 = 0 have?

Answers

Answer:

Zero

Step-by-step explanation:

Because when you replace x with a number and solve it it doesn't have the same answer as x2 − 9 = 0.

I hope this helped. I am sorry if you get this wrong.

The correct answer is 0


3/5 of a juice drink is made of real juice. What percent of the drink is
real juice?​

Answers

60% is made out of real juice

Answer:

60%

Step-by-step explanation:

Percent means out of 100

Changing 3/5 to a denominator of 100

3/5*20/20

60/100

The percent is 60 %

What is the size of angle YXZ?

Answers

We have a right triangle and we're given the leg lengths.  

tan X = |YZ| / |XZ| = 20/8 = 5/2

X = arctan(5/2) = 68.2°

Answer: 68.2°

tan X = YZ / XZ = 20/8 = 5/2

X = arctan(5/2) = 68.2°

Answer: 68.2°

What is the surface area of a hemisphere with a radius 10

Answers

Answer:

Maths keeps one mentally active. The total surface of a hemisphere = 3(pi)r^2. So if the radius = 10 cm, then the TSA = 3(pi)r^2 = 300(pi) = 942.8571429 sq cm.

Step-by-step explanation:

hope this helps you :)

Answer:

The total surface of a hemisphere = 3(pi)r^2.

So if the radius = 10 cm, then the TSA = 3(pi)r^2 = 300(pi) = 942.8571429 sq cm.

which set of sides make a right triangle

Answers

Answer:

A right triangle consists of two legs and a hypotenuse.

Step-by-step explanation:

The two legs meet at a 90° angle and the hypotenuse is the longest side of the right triangle and is the side opposite the right angle. There are a couple of special types of right triangles, like the 45°-45° right triangles and the 30°-60° right triangle.

4x-y+ 2z=-1

Given the system -x+2y + 5z = 2, which is true?

|-x+y-3z= 1

Answers

Answer:

Y = 0

X= 1/2

Z = -1/2

Step-by-step explanation:

4x-y+ 2z=-1

-x+y-3z= 1

-x+2y + 5z = 2

Solving simultenously

Y= 4x + 2z -1

Y =1+ 3z+ x

Y =x/2 -( 5z/2) - 1

Equating y will give two equations

3x-z = 2

3x + 11z = -4

Subtracting the equations

-12z =6

Z= -1/2

Substituting z

3x +1/2 = 2

3x = 3/2

X= 1/2

Substituting x and z to find y in

-x+y-3z= 1

-1/2 + y +3/2 = 1

Y = 1-1

Y = 0

Answer: b) is answer

Step-by-step explanation:

To solve VX +VX-5 = 5 for x, begin with which of these steps?

Answers

Answer:

x = 5/v

Step-by-step explanation:

Solve for x:

2 v x - 5 = 5

Add 5 to both sides:

2 v x = 10

Divide both sides by 2 v:

Answer: x = 5/v

Answer:

I'd say start with "Add 5 to both sides"

Step-by-step explanation:

VX +VX-5 = 5

Add 5 to both sides

2VX=10

Divide both sides by 2

VX=5

Divide both sides by V

X=[tex]\frac{5}{V}[/tex]

Making Friends Online
A survey conducted in March 2015 asked 1060 teens to estimate, on average, the number of friends they had made online. while 43% had not made any friends online, a small number of the teens had made many friends online.
(a) Do you expect the distribution of number of friends made online to be symmetric, skewed to the right, or skewed to the left?
Skewed to the left.
Symmetric
Skewed to the right.
(b) Two measures of center for this distribution are 1 friend and 5.3 friends. Which is most likely to be the mean and which is most likely to be the median?
Mean=
Median=
1Lenhart A, "Teens, Technology, and Friendships", Pew Research Center, pewresearch.org, August 6, 2015. Value for the mean is estimated from information given.

Answers

Answer:

Step-by-step explanation:

) Do you expect the distribution of number of friends made online to be symmetric, skewed to the right, or skewed to the left?

Skewed to the left.

Symmetric

Skewed to the right.

(b) Two measures of center for this distribution are 1 friend and 5.3 friends. Which is most likely to be the mean and which is most likely to be the median?

Mean=

Median=

----------------------------------

a)

as proportion of people with 0 friends is 43% whcih is on left side and maximum ; and % decrease with increasing number of friends

skewed to the right

b)

as it is skewed to the right ; therefore mean is greater than median

mean=5.3

median=1

[since for skwewed to the right distribution :mean is always greater than median, therefore higher value should be mean which is 5.3 and lower value is median which is 1]

Part(a): Skewed to the right

Part(b) The required values are,

mean=5.3

median=1

a)

As a proportion of people with 0 friends are 43% which is on the left side and maximum; and % decrease with an increasing number of friends

skewed to the right

b)

As it is skewed to the right; therefore mean is greater than the median

mean=5.3

median=1

Learn More:https://brainly.com/question/11729451

Assume that the weight loss for the first month of a diet program varies between 6 pounds and 12 pounds, and is spread evenly over the range of possibilities, so that there is a uniform distribution. Find the probability of the given range of pounds lost. Between 8 pounds and 11 pounds.
A. 1/2.B. 1/4.C. 2/3.D. 1/3.

Answers

Answer:

A. 1/2

Step-by-step explanation:

In this uniform distribution (from 6 to 12 pounds), the probability of any given range (from 'a' to 'b' pounds) is determined by:

[tex]P = \frac{b-a}{12 - 6}[/tex]

For a = 8 pounds and b = 11 pounds, the probability is:

[tex]P=\frac{11-8}{12-6}\\P=\frac{3}{6}=\frac{1}{2}[/tex]

The probability of the range of pounds lost being between 8 pounds and 11 pounds is 1/2.

A car rental company charges a daily rate of ​$35 plus ​$0.20 per mile for a certain car. Suppose that you rent that car for a day and your bill​ (before taxes) is ​$97. How many miles did you​ drive?

Answers

Answer:

360 miles

Step-by-step explanation:

97= 25+0.2m0.2m= 97-250.2m= 72m= 72/0.2m= 360 miles

need answers to 30 and 31

Answers

Answer:

C ; A

Step-by-step explanation:

Question 30:

Perimeter is the sum of all sides.

Perimeter for a recatngle can be found with the formula:

2(L+W)

Length is 7

Width is 4

Plug our values in.

2(7+4)

2(11)

22

Answer C

Question 31:

Circumference of a circle can be found with the formula:

πd.

Diameter of the given circle is 6.

Plug it in

Round π to 3.14

6(3.14)

18.84

Answer A

Which point is an x-intercept of the quadratic function f(x) = (x + 6)(x – 3)? (0,6) (0,–6) (6,0) (–6,0)

Answers

Answer: (-6, 0)

Step-by-step explanation:

X-intercepts of equations are any points on the equation that lie on the x-axis, or the horizontal line "y = 0".

In order to find the x-intercept of an equation, find the points that will satisfy the equation "y = 0":

y = (x + 6)(x - 3)

y = 0

(x + 6)(x - 3) = 0

With this equation, you can find which points lie on the x-axis.

When x = -6, the equation is: 0 * -9 = 0, which is correct.

When x = 3, the equation is 9 * 0 = 0, which is correct.

Make sure you're picking the correct coordinate out of the answer choices.

The x-coordinates are -6 and 3, and the y-coordinates are 0, because the points lie on the x-axis.

The correct answer is (-6, 0).

(3, 0) is also correct, but the question does not require it.

Answer:

D

Step-by-step explanation:

9. The mean is defined as the
A. number that shows up most often in a data set.
B. average of a data set.
C. middle of the data set.
D. range of the data set.

Answers

Answer:

B. Average of the data set

Step-by-step explanation:

The mean is defined as the average of a data set and it's formula is

Mean = [tex]\frac{sum of observations}{number of observations}[/tex]

Fertilizer: A new type of fertilizer is being tested on a plot of land in an orange grove, to see whether it increases the amount of fruit produced. The mean number of pounds of fruit on this plot of land with the old fertilizer was 380 pounds. Agriculture scientists believe that the new fertilizer may change the yield. State the appropriate null and alternative hypotheses.

Answers

Answer:

The null and alternative hypothesis for this problem are:

[tex]H_0:\mu=380\\\\H_a: \mu>380[/tex]

Step-by-step explanation:

The alternative hypothesis shows the claim of the researchers. In this case, that the new type of fertilizer significantly increase the actual yield with the old fertilizer:

[tex]H_a: \mu>380[/tex]

The null hypothesis is the hypothesis to be nullified, so it states that the claim is not true and the yield is the same (or, at least, not significantly higher) as with the old fertilizer:

[tex]H_0: \mu=380[/tex]

Which of the following is the perimeter of a triangle with side lengths of 18 cm, 26 cm, and 32 cm?

Answers

Answer:

76 cm

Step-by-step explanation:

To find the perimeter, add up all of the side lengths.

18 cm + 26 cm + 32 cm = 76 cm

I hope this helps :))

Which sequences are geometric? Check all that apply.
O 1,5, 25, 125, ...
3, 6, 9, 12,...
3, 6, 12, 24, ...
3, 9, 81, 6, 561, ...
10, 20, 40, 60, ...

Answers

Answer:

1, 5, 25, 125, ...

3, 6, 12, 24, ...

Step-by-step explanation:

a geometric progression is a sequence of numbers where each term after the first is found by multiplying the previous one by a fixed, non-zero number called the common ratio

1, 5, 25, 125, ...

yes, the common ratio is 5

3, 6, 9, 12,...

no

3, 6, 12, 24, ...

yes, the common ratio is 2

3, 9, 81, 6, 561, ...

no

10, 20, 40, 60, ...

no

Some cruise ship passengers are given magnetic​ bracelets, which they agree to wear in an attempt to eliminate or diminish the effects of motion sickness. Others are given similar bracelets that have no magnetism. What type of study is​ this? What are the variables of​ interest?
Choose the correct answer below.
A. Observational study. The variable of interest is whether the passenger experienced motion sickness.
B. Observational study. The variable of interest is whether a passenger's bracelet is magnetized or not.
C. Experiment. The variable of interest is whether the passenger experienced motion sickness.
D. Experiment. The variable of interest is whether a passenger's bracelet is magnetized or not.

Answers

Answer:

Option c

Step-by-step explanation:

This is an experiment because the researcher wants to test efficiency of the magnetic bracelets in the elimination of motion sickness i.e. whether they experienced motion sickness even after wearing the magnetic bracelets.

Other Questions
identify the domain and range of y = 4x - 5 + 3 Which two laws were discovered by Antoine Lavoisier and Joseph Proust g One of the dangers of studying intercultural communication is _______________, or presuming you understand an individual because of your knowledge about their cultural group. Group of answer choices Why did nationalism backfire for Bismarck?A: The economy was dismal, and the workers blamed Bismarck for it.B: The creation of the Triple-Alliance caused nationalism to backfire.C: They lost the Franco-Prussian war.D: There was feuding between Bismarck and Wilhelm II. Dana is looking for images that illustrate the contrast principle of design in an advertising layout. Which images should she pick? What is the reciprocal of 8/12 PLZ HLP MEEEEEEEEEEEEEEEthe table below shows what 300 boxer students ate for lunch on friday. what is the ratio of the number of students who ate soup to the number of students who did not eat soup? Find the Surface Area of this triangular prism. Find the surface area of this triangular prism. Be sure to include the correct unit in your answer.15 m12 m09 m11 mThanks for anyone that answers Here's a graph of a linear function. Write theequation that describes that function.Express it in slope-intercept form.HELP PLEASE Which evidence would most strongly indicate that a chemical change istaking place?A. FizzingB. Changing shapeC. BreakingD. Changing state A non-standard (but still fair) die has sides 2, 3, 5, 8, 12, 15, 18 and 20. What is P(roll an odd number)? the ratio of the number of Annes pencils to the number of jasons pencils is 4:3. Anne has 100 pencils how many pencils does Jason have 5. If there is a large number of prey rodents, the number of owls will increase due to a healthyfood source. More owls feeding on the rodents will cause their population to decrease, which inturn will eventually cause the owls number to decrease. Would you expect to find this pattern in other ecosystems between predators and prey? Explain. The Fast Freight Shipping Company charges the following rates for different package weights: 2 pounds or less: $1.50 over 2 pounds but not more than 6 pounds: $3.00 over 6 pounds but not more than 10 pounds: $4.00 over 10 pounds: $4.75 Write a program that asks the user to enter the weight of a package and then displays the shipping charges. The program should also do "Input Validation" that only takes positive input values and show a message "invalid input value!" otherwise. Following World War I, ___________ were areas of the Middle East and North Africa that were administered by a European power with the aim of preparing the population for self-government and future independence. Break the word "already" into syllables, use the symbol (/) to separateword *Your answer A 2500 kg truck moving at 10.00 m/s strikes a car waiting at the light. Assume there is no friction on the road. The hook bumpers continue to move at 7.00 m/s. What is the mass of the struck car Which equation can you use to calculate themissing credit score?*-7801.0728x - 28780=1.0728 =x-1.07780 a research submarine what is the maximum depth it can go